Gönderen Konu: Diyafont Denklemler Çalışma Soruları ($138$ Tane)  (Okunma sayısı 18143 defa)

Çevrimdışı AtakanCİCEK

  • G.O Demirbaş Üye
  • ******
  • İleti: 264
  • Karma: +4/-0
  • Manisa
Ynt: Diyafont Denklemler Çalışma Soruları (70 Tane)
« Yanıtla #75 : Ağustos 18, 2019, 08:01:59 ös »
$44)$

$n=3$  için $(1,1)$ bir çözümdür.

$7x_n^2+y_n^2=2^n$  denkleminin sağlandığını kabul edelim.  Öyle ki  $7X_n^2+Y_n^2=2^{n+1}$  çözümü bulunduğunu gösterelim.
 
$$7.(\dfrac{x_n\pm y_n}{2})^2+(\dfrac{7x_n\mp y_n}{2})^2=2.(7x_n^2+y_n^2)=2^{n+1}$$  Olduğundan dolayı $x_n+y_n$  çift olacak şekilde  bir çözüm daima vardır. $n=3$ için $(1,1)$ çözümü olduğundan $n=4$ için bu özelliği kullanarak örneğin $(0,4)$  ya da $(1,3)$ çözümlerini görürüz.
Bir matematikçi sanmaz fakat bilir, inandırmaya çalışmaz çünkü ispat eder.
    Boğaziçi Üniversitesi - Matematik

Çevrimdışı AtakanCİCEK

  • G.O Demirbaş Üye
  • ******
  • İleti: 264
  • Karma: +4/-0
  • Manisa
Ynt: Diyafont Denklemler Çalışma Soruları (70 Tane)
« Yanıtla #76 : Ağustos 18, 2019, 08:27:54 ös »
$42)$

$p^mq^n=(p+q)^2+1=p^2+2pq+q^2+1$ olduğu için $p\mid q^2+1$  ve $q\mid p^2+1$  olduğu açıktır.

$p=q$ olduğunu düşünelim. $p\mid p^2+1$ olur. $p\mid1$ çıkar ki bu mümkün değildir.

Genelliği bozmadan $p<q$ alalım. $p=1$  durumu $q=2$  durumunu gerektirir ki mümkün değildir. $q>p\ge2$

$$p^mq^n=(p+q)^2+1<4q^2<p^2q^2<pq^3\le p^mq^3$$ eşitsizliklerinden $n<3$  olduğu açıktır.

$n=2$  için $p^m<4$ olur. $m=1$ , $p=2$  veya $p=3$ olduğunu kullanalım. 

   $p=2$ için $$2q^2=(2+q)^2+1$$ olur ve bu $q=5$ için çözümdür.

   $p=3$ için $$3q^2=(3+q)^2+1$$ olur çözümü yoktur.

$n=1$  için  $p^m<4q$ gelir.  $q\mid p^2+1$ olduğunu biliyoruz. 

$q=p^2+1$ ise $p\mid q^2+1=p^4+2p^2+2$ bu da $p\mid2$ olduğunu gösterir. $p=2$  olduğunu gösterir bu durumda $p=5$ bulunur. Sağ taraf $25$ ile bölünürken sol taraf $5$ ile bölüneceğinden çözüm yoktur.

Bu nedenle $q\le \dfrac{p^2+1}{2}$  olur.

$p^m<2.(p^2+1)$ elde  edilir.  $p=2$ için $m\le 3$ ,  Diğer durumlarda $m\le2$ bir çözümdür.

$p=2$ olması demek $q\le \dfrac{5}{2}$ olması demektir.  fakat $q>p$ den dolayı çelişki gelir.

$m\le2$ olduğu bulunur.

   $m=1$ için $$pq=(p+q)^2+1$$ olur  çözüm gelmeyeceği açıktır.

   $m=2$  için $p^2<4q$  olur. $q\mid p^2+1$  olduğu da bilindiğinden $p^2+1=q,2q,3q,4q,$ olasılıkları ortaya çıkar.

    $p^2+1$ ifadesi $(mod3)$ veya $(mod4)$  te $0$ kalanı vermeyeceğinden ve $p^2+1=q$ durumunu zaten incelediğimiz için
     
     $p^2+1=2q$ olur. $p^4+2p^2+5=4q^2+4$ olur.

      $p\mid q^2+1$ olduğundan $4p\mid p^4+2p^2+5$ elde edilir. Buradan $p\mid 5$ gelir. $p=5$ bulunur.  $n=1$ $m=2$  durumu olduğunu da         

       göz önüne alırsak $q=13$ çözümü gelir. 

     Şimdi denklemin tüm çözümlerini sıralayabiliriz.

       $$(m,n,p,q)\in \{(2,1,5,2),(2,1,5,13),(1,2,2,5),(1,2,13,5)\}$$
Bir matematikçi sanmaz fakat bilir, inandırmaya çalışmaz çünkü ispat eder.
    Boğaziçi Üniversitesi - Matematik

Çevrimdışı AtakanCİCEK

  • G.O Demirbaş Üye
  • ******
  • İleti: 264
  • Karma: +4/-0
  • Manisa
Ynt: Diyafont Denklemler Çalışma Soruları (70 Tane)
« Yanıtla #77 : Ağustos 18, 2019, 10:12:39 ös »
$72)$  Scarface hocamın youtubede paylaştığı çözümü yazıyorum.

$(0,0)$ çözümü açıktır.

artık $m>0$  olmalıdır.

$7n^2=m^3+15m$  olduğundan  denkleme $7$  modunda bakılırsa $m\equiv0(mod7)$  olduğu görülür.



Düzenlenirse $n^2=k.(49k^2+15)$ olur.

$(k,49k^2+15)=(k,15)=\{1,3,5,15\}$ olmalıdır.

$(k,49k^2+15)=1$ ise  $49k^2+15=z^2$ 

\begin{equation*}
\begin{cases}
z-7k=1,
\\
z+7k=15
\end{cases}
\end{equation*}

çözülürse $k=1$  $m=7$  gelir. $(7,8)$ ,$(7,-8)$  gelir.

$(k,49k^2+15)=3$  olursa

$k=3t$  denilip dönüşüm yapılınca

$n^2=9t.(147t^2+15)$  ayrı ayrı tamkare olmalıdırlar.  $z^2=147t^2+15$  $z^2\equiv2(mod3)$  ten çelişki gelir.

$(k,49k^2+15)=5$  olursa

$n^2=25.t.(49.5t^2+3)$  ayrı ayrı tamkare olmalıdırlar. $z^2=49.5t^2+3$  $z^2\equiv 3(mod5)$

$(k,49k^2+15)=15$  olursa

$n^2=225.t.(49.15t^2+1)$ ayrı ayrı tam kare olmalıdır.   

$49.15t^2+1=z^2$  olsun.   $z^2-735t^2=1$ pell denklemi elde edilir.  Sürekli Kesir açınımları yöntemi kullanılırsa $1$  pozitif tam sayı çözümü

bulunacaktır. Aynı zamanda $(m,n)$ çözüm ise $(m,-n)$  de bir çözüm olacağından $2$ farklı çözümü vardır.

Yani toplam $1+2+2=5$ çözümü vardır.
Bir matematikçi sanmaz fakat bilir, inandırmaya çalışmaz çünkü ispat eder.
    Boğaziçi Üniversitesi - Matematik

Çevrimdışı Metin Can Aydemir

  • G.O Genel Moderator
  • Geo-Maniac
  • ********
  • İleti: 1139
  • Karma: +9/-0
Ynt: Diyafont Denklemler Çalışma Soruları (70 Tane)
« Yanıtla #78 : Ağustos 20, 2019, 02:37:05 ös »
$70)$ Denklemi birkaç durumda inceleyeceğiz ama önce $m=0,1,2,3,4$ için incelersek $(m,n)=(0,-1),(2,17)$ çözümlerini elde ederiz.

$i)$ $m\geq 5$ ise,

$ia)$ $m\geq n$ ise, $$m^3(m-2)+m\geq n^3(m-2)+n=m^4+1\Rightarrow 2m^3+1\leq m$$ olur. Çelişki

$ib)$ $n\geq m+1$ ise, $$m^4+1=n^3(m-2)+n\geq (m+1)^3(m-2)+(m+1)\Rightarrow 0\geq m^3-3m^2-4m-2$$ olur. $m^3-3m^2-4m-2$ fonksiyonu $m\geq 5$ için pozitiftir. Çelişki

$ii)$ $m<0$ ise, $n=0$ için çözüm yoktur.

$iia)$ $n>0$ ise $mn^3+n\leq 0$ fakat $m^4+2n^3+1>0$ olacağından çözüm gelmez.

$iib)$ $n<0$ ise, $m=-a$, $n=-b$ olsun. $a,b>0$ ve $$a^4-2b^3+1=ab^3-b$$ olur. $a=1$ için $b=1$ olur ve buradan $(m,n)=(-1,-1)$ çözümü gelir. $a>1$ için, $$f(b)=b^3(a+2)-b-(1+a^4)$$ olsun. $f'(b)=3b^2(a+2)-1>0$ olur. Yani $f$ fonksiyonu artandır. $$f(a)=2a^3-a-1>0$$ ve $$f(a-1)=-a^3-3a^2+4a-2<0$$olur. Artan fonksiyon olduğu için $f$ fonksiyonun tek çözümü $(a-1,a)$ aralığında olması gerekir fakat tamsayı olamaz. Dolayısıyla buradan çözüm gelmez.

Tüm çözümler $(m,n)=(0,-1),(2,17),(-1,-1)$'dir.
Gerçek hikayeler aslında söylenmeyenlerdir.

Çevrimdışı Metin Can Aydemir

  • G.O Genel Moderator
  • Geo-Maniac
  • ********
  • İleti: 1139
  • Karma: +9/-0
Ynt: Diyafont Denklemler Çalışma Soruları (70 Tane)
« Yanıtla #79 : Ağustos 21, 2019, 03:12:48 ös »
$75)$ $x=y$ ise $(x,y)=(0,0)$ bulunur. $x\neq y$ ise $x>y$ olmalıdır. $x-y=k$ olsun. $k>0$'dır. $$(y+k)^2+y^2=k^3$$ denklemini düzenlersek, $$(2y+k)^2=k^2(2k-1)$$ olur. Burada $2k-1$ tamkare olmalı. $2k-1=(2t+1)^2$ olsun, $k=2t^2+2t+1$ yazarsak, $$(2t^2+2t+1)^2(2t+1)^2=(2y+2t^2+2t+1)^2$$ Buradan $y=2t^3+2t^2+t$ veya $-2t^3-4t^2-3t-1$ bulunur. $y+k=x$'den,

Tüm çözümler, $(x,y)=(0,0),(2t^3+4t^2+3t+1,2t^3+2t^2+t),(-2t^3-2t^2-t,-2t^3-4t^2-3t-1)$'dir.
Gerçek hikayeler aslında söylenmeyenlerdir.

Çevrimdışı Metin Can Aydemir

  • G.O Genel Moderator
  • Geo-Maniac
  • ********
  • İleti: 1139
  • Karma: +9/-0
Ynt: Diyafont Denklemler Çalışma Soruları (70 Tane)
« Yanıtla #80 : Ağustos 21, 2019, 03:30:55 ös »
$76)$ $x=0$ ise $y=\pm 1$

$x>0$ ise $$(x+1)^4>x^4+x^3+x^2+x+1=y^4>x^4$$ olduğundan çözüm yoktur.

$x=-1$ için $y=\pm 1$ olur.

$x<-1$ ise $x=-a$ olsun. $a>1$'dir. $$a^4>y^4=a^4-a^3+a^2-a+1>(a-1)^4$$ olduğundan çözüm yoktur.

Tüm çözümler $(x,y)=(0,1),(0,-1),(-1,1),(-1,-1)$'dir.
Gerçek hikayeler aslında söylenmeyenlerdir.

Çevrimdışı Metin Can Aydemir

  • G.O Genel Moderator
  • Geo-Maniac
  • ********
  • İleti: 1139
  • Karma: +9/-0
Ynt: Diyafont Denklemler Çalışma Soruları (70 Tane)
« Yanıtla #81 : Ağustos 23, 2019, 01:34:15 ös »
$74)$ $x=0,1,2,3,4,5$ için denersek $(x,y)=(3,1),(5,3)$ çözümleri gelir. $x\geq 6$ için $3^y+5\equiv 0(mod~64)$ olur. Bu şartı sağlayan $y$ değeri için $y\equiv 11(mod~16)$'dir. $$2^x\equiv 3^{16k+11}+5\equiv 3^{11}+5\equiv 12 (mod~17)$$ $$\Rightarrow 2^x\equiv 12 (mod~17)$$ fakat hiçbir $x$ değeri için bu sağlanamaz. Dolayısıyla tüm çözümler $(x,y)=(3,1),(5,3)$'dir.
Gerçek hikayeler aslında söylenmeyenlerdir.

Çevrimdışı Metin Can Aydemir

  • G.O Genel Moderator
  • Geo-Maniac
  • ********
  • İleti: 1139
  • Karma: +9/-0
Ynt: Diyafont Denklemler Çalışma Soruları (70 Tane)
« Yanıtla #82 : Ağustos 27, 2019, 04:10:17 ös »
$42)$ $b=0$ için $k=-a^2$ olur fakat $k\geq 0$ olduğundan $a=k=0$ olabilir. Benzer şekilde $a=0$ için de aynı durum gelir. $a=b>0$ için, $$k=\dfrac{3a^2}{a^2-1}=3+\dfrac{3}{a^2-1}$$ olur. Buradan tek çözüm $a=2$, $k=4$'dür. Şimdi genelliği bozmadan $a>b$ olsun.

$i)$ $b=1$ ise $$k=\dfrac{a^2+a+1}{a-1}=a+2+\dfrac{3}{a-1}$$ olur. Buradan $a=2$ ve $a=4$ çözümleri gelir ve her ikisi için de $k=7$'dir.

$ii)$ $a>b>1$ ise $$k=\dfrac{a^2+b^2+ab}{ab-1}\geq \dfrac{3ab}{ab-1}>3$$ olduğundan $k\geq 4$ olur. Ayrıca $k=4$'ün sağladığını zaten biliyoruz. Dolayısıyla $k>4$ için incelememiz yeterlidir. Şimdi bu fonksiyonu ve $m>n>1$ şartını sağlayan tamsayı çiftlerinin arasından en küçük $m$ değerli olanı alalım. $$f(x)=x^2-(k-1)nx+(n^2+k)$$ olsun. $m$ bu fonksiyonun bir çözümüdür. $n^2\geq 4$ olduğundan $$f(n)=k-(k-3)n^2<0$$ olur. Eşit olamaz çünkü eşit olursa $$n^2(k-3)=k \Rightarrow k=n^2(k-3)\geq 4k-12\Rightarrow k=4$$ olur. Çelişki.

$f$ fonksiyonu bir paraboldür ve kolları yukarı doğru bakar. Dolayısıyla $f(n)$ negatifse $n$ iki kök arasında olmalıdır. Diğer kök $p$ olsun. $m>n>p$'dir. Vieta teoreminden $$m+p=(k-1)n$$ $$mp=n^2+k$$ buradan $p$ pozitif tamsayı bulunur. Dolayısıyla $(m,n,k)$ çözüm ise $(n,p,k)$ da bir çözümdür ve $m$'nin en küçük çözüm olmasıyla çelişir. Dolayısıyla $p=1$ olmalıdır. Buradan da daha önce bulduğumuz çözümler bulunur. Yani tüm çözümler $k=0,4,7$ değerlerini alabilir.
Gerçek hikayeler aslında söylenmeyenlerdir.

Çevrimdışı Metin Can Aydemir

  • G.O Genel Moderator
  • Geo-Maniac
  • ********
  • İleti: 1139
  • Karma: +9/-0
Ynt: Diyafont Denklemler Çalışma Soruları (70 Tane)
« Yanıtla #83 : Ağustos 27, 2019, 04:40:18 ös »
$31)$ $xy(x^2+y^2)=\dfrac{(x+y)^4-(x-y)^4}{8}=2z^2$ Buradan $$(x-y)^4+(4z)^2=(x+y)^4$$ olur. Fermat'ın son teoreminden bu denklemin çözümü için $(x-y)=0$ veya $z=0$ olmalıdır. $a^4+b^2=c^4$ denkleminin çözümünün olmadığının ispatı için "Proof of Fermat's Last Theorem for specific exponents" 'ı incelemeniz yeterlidir.

$i)$ $z=0$ için $xy(x^2+y^2)=0$ olur. Buradan $x=0$ veya $y=0$ bulunur. $(x,y,z)=(0,k,0),(k,0,0)$ çözümleri gelir.

$ii)$ $x=y$ ise $2z^2=x^2(x^2+x^2)=2x^4$ buradan da $z=\pm x^2$ bulunur. Buradan $(x,y,z)=(k,k,k^2),(k,k,-k^2)$ çözümleri bulunur.

Tüm çözümler $(x,y,z)=(0,k,0),(k,0,0),(k,k,k^2),(k,k,-k^2)$'dir.
Gerçek hikayeler aslında söylenmeyenlerdir.

Çevrimdışı Metin Can Aydemir

  • G.O Genel Moderator
  • Geo-Maniac
  • ********
  • İleti: 1139
  • Karma: +9/-0
Ynt: Diyafont Denklemler Çalışma Soruları (70 Tane)
« Yanıtla #84 : Eylül 11, 2019, 05:00:49 öö »
$77)$ Verilen ifadeyi $p$ modunda inceleyelim. $$p^3+mp+2m\equiv m^2+p+1~(\text{mod}~p)\Rightarrow (m-1)^2\equiv 0~(\text{mod}~ p)$$ olur. Şimdi $m=pk+1$ olsun. $m=1$ için çözüm gelmediğinden $k$ da pozitiftir. Yerine yazarsak $$p^3+(pk+1)(p+2)=(pk+1)^2+p+1\Rightarrow p=k^2-k$$ bulunur. Yani $p$ çifttir. $p=2$ dersek $k=2$ bulunur. $m=2\cdot 2+1=5$ olur.

Tek çözüm, $(m,p)=(5,2)$'dir.
Gerçek hikayeler aslında söylenmeyenlerdir.

Çevrimdışı AtakanCİCEK

  • G.O Demirbaş Üye
  • ******
  • İleti: 264
  • Karma: +4/-0
  • Manisa
Ynt: Diyafont Denklemler Çalışma Soruları (70 Tane)
« Yanıtla #85 : Ocak 01, 2020, 08:58:24 ös »
$79)$
$$(a^2+1).(b^2+1).(c^2+1)=a^2b^2c^2+a^2b^2+a^2c^2+b^2c^2+a^2+b^2+c^2+1$$

$$(ab+ac+bc)^2=a^2b^2+a^2c^2+b^2c^2+2.abc.(a+b+c)$$

$$(a+b+c)^2=a^2+b^2+c^2+2.(ab+ac+bc)$$

$$a^2b^2c^2+1-2abc.(a+b+c)+(a+b+c)^2-2+1=(a+b+c-abc)^2=3^x-5^y$$ bulunur.

$a+b+c-abc=p$ dönüşümü yapalım.

$p^2=3^x-5^y$

$p^2\equiv \{0,1,4\}(mod5)\equiv 3^x(mod5)$  $3^x$  ifadesi $0,1,4$ kalanlarından birini vermesi için $x$ in çift olduğu açıktır.

$x=2t$ , $t\in N$  dönüşümü yapılabilir. İki kare farkından

$$(p-3^t).(p+3^t)=-5^y$$

Varsayalım ki  $p+3^t\le 0$ olsun.  O halde $p-3^t \ge 0$ olur. Buradan $p-3^t\ge p+3^t$ yani $3^t\le  0$ elde edilir. Bu bir çelişkidir.
O halde $p+3^t$ pozitiftir.

\begin{equation*}
\begin{cases}
p+3^t=5^g
\\p-3^t=-5^{y-g}
\end{cases}
\end{equation*}
, $g \in N$ dönüşümü yapılabilir. Düzenlersek  $$2.3^t=5^g+5^{y-g}$$ elde edilir. Denklemin solu $5$ ile bölünemeyeceğinden sağ kısmı da bölünememelidir. $g=0$ dır veya $y=g$ dir.

$1)$ $g=0$ için $2.3^t=5^y+1$ Lifting The Exponent Lemma'nın $2.$ formunu uygulayalım.

$3\mid 5+1 $ olduğundan $v_3(2.3^t)=v_3(5+1)+v_3(g)$ Buradan $g=k.3^{t-1}$ , $k \in N$ dönüşümü yapabiliriz.

Yerine koyarsak $2.3^t=5^{k.3^{t-1}}+1$ olduğundan $t>1$ için çelişki gelir.

    a) $t=0$ için $y=0$ elde edilir. Buradan $p=0$ elde edilir.Fakat $$(a^2+1).(b^2+1).(c^2+1)=p^2\ge 1$$ olduğu için mümkün değildir.

   b)   $t=1$  için $y=1$  elde edilir. $p=-2$ elde edilir.

2)   a) $y=g$  ve $t=0$ için $y=0$ bulunur. $1)$ in $a$  seçeneği ile aynıdır, mümkün değildir.

       b) $y=g$ ve $t=1$ için $y=1$ bulunur. $p=2$ elde edilir.

Elimizde $2$ adet denklem sistemi oluştu

\begin{equation*}
\begin{cases}
ab+ac+bc=1
\\
abc-(a+b+c)=2
\end{cases}
\end{equation*}

\begin{equation*}
\begin{cases}
ab+ac+bc=1
\\
abc-(a+b+c)=-2
\end{cases}
\end{equation*}

Denklemlerde $c=\dfrac{1-ab}{a+b}$ yazarsak

$\dfrac{ab-a^2b^2}{a+b}-\dfrac{(a+b)^2+1-ab}{a+b}=\dfrac{a^2b^2+a^2+b^2+1}{a+b}=\pm 2$

$(ab)^2+(a\pm 1)^2+(b\pm 1)^2=1$ elde edilir. Buradan karelerin birini $1$ e diğer ikisini $0$ a eşitlersek

$(1,1,0),(1,0,1),(0,1,1),(-1,-1,0),(-1,0,-1),(0,-1,-1)$ şeklinde $6$ çözüm elde edilir.
« Son Düzenleme: Ocak 04, 2020, 10:09:42 ös Gönderen: AtakanCİCEK »
Bir matematikçi sanmaz fakat bilir, inandırmaya çalışmaz çünkü ispat eder.
    Boğaziçi Üniversitesi - Matematik

Çevrimdışı Metin Can Aydemir

  • G.O Genel Moderator
  • Geo-Maniac
  • ********
  • İleti: 1139
  • Karma: +9/-0
Ynt: Diyafont Denklemler Çalışma Soruları ($138$ Tane)
« Yanıtla #86 : Ekim 20, 2020, 02:43:39 ös »
$80)$ Verilen ifadeyi düzenlersek, $$n^2=(x^2+1)(y^2+1)+2(x-y)(1-xy)-4xy=(x+1)^2(y-1)^2$$ bulunur. Buradan $n=(x+1)(y-1)$ veya $-n=(x+1)(y-1)$ bulunur. $n$'yi bölen her $a$ tamsayısı için $(x,y)=\left (a-1,\dfrac{n}{a}+1\right ),\left (a-1,-\dfrac{n}{a}+1\right )$ çözümleri bulunabilir. Bu çözümler çakışık olamaz çünkü olsaydı $\dfrac{1}{a}=0$ olur fakat bu sağlanamaz. Her $a$ böleni için denklemin tam olarak $2$ çözümü bulunur. Dolayısıyla toplam çözüm sayısı, $n$'nin bölen sayısının $2$ katıdır.
Gerçek hikayeler aslında söylenmeyenlerdir.

Çevrimdışı Metin Can Aydemir

  • G.O Genel Moderator
  • Geo-Maniac
  • ********
  • İleti: 1139
  • Karma: +9/-0
Ynt: Diyafont Denklemler Çalışma Soruları ($138$ Tane)
« Yanıtla #87 : Ekim 20, 2020, 02:52:20 ös »
$81)$ $\dfrac{1}{x}+\dfrac{1}{y}=\dfrac{1}{n}$ olduğundan $\dfrac{1}{n}>\dfrac{1}{x}$ ve $\dfrac{1}{n}>\dfrac{1}{y}$ olmalıdır. Buradan $x>n$ ve $y>n$ bulunur. Şimdi denklemi düzenlersek, $$\dfrac{x+y}{xy}=\dfrac{1}{n}\Rightarrow xy=nx+ny\Rightarrow (x-n)(y-n)=n^2$$ bulunur. $x>n$ ve $y>n$ olduğundan $(x-n)$ ve $(y-n)$ ifadeleri pozitiftir. $n^2$'nin her pozitif $a$ böleni için $(x,y)=\left ( a+n,\dfrac{n^2}{a}+n \right )$ çözümü bulunur. Dolayısıyla toplam çözüm sayısı $n^2$'nin pozitif bölen sayısıdır.
Gerçek hikayeler aslında söylenmeyenlerdir.

Çevrimdışı Metin Can Aydemir

  • G.O Genel Moderator
  • Geo-Maniac
  • ********
  • İleti: 1139
  • Karma: +9/-0
Ynt: Diyafont Denklemler Çalışma Soruları ($138$ Tane)
« Yanıtla #88 : Ekim 20, 2020, 03:42:14 ös »
$82)$ Denklem simetrik olduğundan genelliği bozmadan $x\geq y$ diyebiliriz. Buradan, $$2x^2\geq x^2+y^2=(xy-9)^2\Rightarrow 0\geq x^2(y^2-2)-18xy+81$$ $$\Rightarrow 0\geq x^2(y^2-2)-18xy+81\geq x^2(y^2-2)-18x^2+81$$ bulunur. Eğer $y\geq 5$ ise $$0\geq x^2(y^2-20)+81\geq 5x^2+81>0$$ bulunur ve bu bir çelişkidir. Dolayısıyla $y\leq 4$ olmalıdır.

$i)$ $y=0$ ise $81=x^2$ elde edilir, buradan $(x,y)=(9,0)$ çözümü gelir.

$ii)$ $y=1$ ise $(x-9)^2=x^2+1$ ve buradan $x=\dfrac{40}{9}$ bulunur. Tamsayı çözümü yoktur.

$iii)$ $y=2$ ise $(2x-9)^2=x^2+4$ ve buradan $3x^2-36x+77=0$ bulunur fakat tamsayı çözümü gelmez.

$iii)$ $y=3$ ise $(3x-9)^2=x^2+9$ ve buradan $4x^2-27x+36=0$ bulunur. Buradan da tamsayı çözümü gelmez.

$iv)$ $y=4$ ise $(4x-9)^2=x^2+16$ ve buradan $15x^2-72x+65=0$ bulunur. Tamsayı çözümü yoktur.

Dolayısıyla tüm çözümler $(x,y)=(9,0),(0,9)$ bulunur.
Gerçek hikayeler aslında söylenmeyenlerdir.

Çevrimdışı Metin Can Aydemir

  • G.O Genel Moderator
  • Geo-Maniac
  • ********
  • İleti: 1139
  • Karma: +9/-0
Ynt: Diyafont Denklemler Çalışma Soruları ($138$ Tane)
« Yanıtla #89 : Ekim 20, 2020, 08:32:10 ös »
$83)$ İfadeyi açalım, $x^2y+xy^2-x^2-y^2=xy(x+y)-(x^2+y^2)=1$ olur. $x^2+y^2=(x+y)^2-2xy$ olduğundan $$xy(x+y)-(x^2+y^2)=xy(x+y)-(x+y)^2+2xy=1\Rightarrow xy(x+y+2)=(x+y)^2+1$$ bulunur. $xy=a$ ve $x+y=b$ için $$a(b+2)=b^2+1\Rightarrow a=\dfrac{b^2+1}{b+2}=b-2+\dfrac{5}{b+2}$$ bulunur. $b+2=\pm 1,\pm 5$ olabilir. Buradan olası $b$ değerleri $\{-7,-3,-1,3\}$ bulunur. Bunlara karşılık gelen $a$ değerlerini bulabiliriz. Olası $(a,b)$ ikilileri $(-10,-7)$, $(-10,-3)$, $(2,-1)$, $(2,3)$ bulunur. $(a,b)=(-10,-3)$ ikilisi için $(x,y)=(-5,2),(2,-5)$ çözümleri bulunur. $(a,b)=(2,3)$ için $(x,y)=(2,1),(1,2)$ çözümleri bulunur. $(a,b)=(-10,-7)$ ve $(a,b)=(2,-1)$ ikilileri için çözüm gelmez. Tüm çözümler, $\boxed{(x,y)=(-5,2),(2,-5),(2,1),(1,2)}$'dir.
Gerçek hikayeler aslında söylenmeyenlerdir.

 


Sitemap 1 2 3 4 5 6 7 8 9 10 11 12 13 14 15 16 17 18 19 20 21 22 23 24 25 26 27 28 29 30 31 32 33 34 35 36 37 
SimplePortal 2.3.3 © 2008-2010, SimplePortal